Download If r = 0 is rational and x is irrational, prove

Survey
yes no Was this document useful for you?
   Thank you for your participation!

* Your assessment is very important for improving the workof artificial intelligence, which forms the content of this project

Document related concepts
no text concepts found
Transcript
Math 5070 Spring 2006 Homework 2: Sets and Logic
Due 1/31/06
Jan Mandel
1. 1/1: If r 6= 0 is rational and x is irrational, prove that r + x and rx are
irrational.
Solution. Suppose r + x is rational, then x = (r + x) − r is rational
because rational numbers form a field, so the difference of two rational
numbers is rational.
Suppose rx is rational, then x = (rx)/r is rational because rational numbers form a field, so the quotient of two rational numbers is rational. The
division is possible because r 6= 0 by assumption. (-1 pt for not using
r 6= 0)
2. 1/4: Let E 6= ∅ be a subset of an ordered set. Suppose α is a lower bound
of E and β is an upper bound of E. Prove that α ≤ β.
Solution. Because α is a lower bound of E,
∀x ∈ E : α ≤ x
and because β is an upper bound of E,
∀x ∈ E : β ≥ x.
Since E 6= ∅, there exists x ∈ E. Then
α ≤ x and x ≤ β
so α ≤ β. (It is OK to write α ≤ x ≤ β. -1pt for not using E 6= ∅)
3. Let A be a nonempty set of real numbers which is bounded below. Let −A
be the set of all numbers −x, where x ∈ A. Prove that inf A = − sup(−A).
Solution. Let a ∈ R. We have
⇐⇒
⇐⇒
⇐⇒
⇐⇒
⇐⇒
⇐⇒
a = − sup(−A)
−a = sup(−A) (multiplying by -1)
(∀x ∈ −A : x ≤ −a) ∧ (∀y < −a∃x ∈ −A : x > y) (from the definition of sup )
(∀ − x ∈ A : x ≤ −a) ∧ (∀y < −a∃ − x ∈ A : x > y) (from the definition of − A)
(∀x ∈ A : −x ≤ −a) ∧ (∀ − y < −a∃x ∈ A : −x > −y) (substitute − x for x and − y for y)
(∀x ∈ A : x ≥ a) ∧ (∀y > a∃x ∈ A : x < y) (multiplying the all inequalities by − 1)
a = inf A (since this is the definition of infimum)
Since A is nonempty and bounded below, inf A exists. Hence inf A =
− sup(−A). (-1 pt for not using that inf A exists.)
1
(a) The assumption that A is nonempty is not needed for the equivalences
above. But all the equivalences prove is that − sup(−A) exists if and
only if inf A does, and if they do, then they equal.
(b) I have of course derived this proof by writing the definitions of a =
− sup(−A) and a = inf A and comparing them; this should be the
starting point of any attempt at the solution.
(c) Another way to structure this proof is: Since A is nonempty and
bounded below, a = inf A exists. Then
a = − sup(−A) =⇒ −a = sup(−A) (multiplying by -1)
etc., replacing all ⇐⇒ above by =⇒.
(d) A more verbal way of writing the same proof: Since A is nonempty
and bounded below, a = inf A exists. Then a is lower bound on
A and no number y > a is a lower bound on A, etc., . . . hence
a = − sup(−A).
4. 1/11 If z ∈ C prove that there exists an r ≥ 0 and w ∈ C with |w| = 1
such that z = rw. Are w and r always uniquely determined by z?
Solution.
Suppose that
z = rw, r ≥ 0, |w| = 1.
Then
|z| = |rw| = |r| |w| = r,
(1)
so r is determined uniquely by r = |z|. If z 6= 0, then dividing z = rw by
r, we get
z |z|
r
z
= = 1.
w = , |w| = =
r
r
|r|
r
So, in conclusion, if z 6= 0, then
r = |z| , w =
z
, r > 0,
r
(2)
and r and w are unique. We need to verify that (2) implies z = rw:
rw = |z|
z
z
= r = z.
r
r
If z = 0, then z = rw with r = 0 and any w ∈ C with |w| = 1; so, in this
case, w is not unique, but r is still unique by (1). (-1 pt for missing the
uniqueness of r)
5. 1/13 If x, y are complex, prove that ||x| − |y|| ≤ |x − y|.
Solution. Using the triangle inequality,
|x| − |y| ≤ |x − y|
2
because |x| + |x − y| = |x| + |y − x| ≤ |x + (y − x)| = |y|. Switching the
notation x and y,
|y| − |x| ≤ |y − x| = |x − y|
which gives
− |x − y| ≤ |x| − |y|
hence
− |x − y| ≤ |x| − |y| ≤ |x − y| ,
so ||x| − |y|| ≤ |x − y|.
6. 11/15 Under what conditions does equality hold in Schwarz inequality?
Solution. From 1.35: With
X
X
X
2
2
A=
|aj | , B =
|bj | , C =
aj bj ,
it was proved that
X
2
2
|Baj − Cbj | = B AB − |C| .
2
(3)
2
Schwarz inequality is |C| ≤ AB. Suppose that AB = |C| . First, if B =
2
0, then (bj ) = 0. Now consider the case B 6= 0. Then, from (3), AB = |C|
P
2
if and only if
|Baj − Cbj | = 0, which implies that Baj − Cbj = 0 for
all j.Dividing by B 6= 0,
C
∀j : aj = bj .
B
So, we have proved
2
AB = |C| =⇒ (bj ) = 0 ∨ ∃t ∈ C : (aj ) = t (bj ) .
(4)
Now we need to show the converse implication. If (bj ) = 0, then B = C =
2
0, so AB = |C| = 0. If (aj ) = t (bj ), then
X
X 2
2
2
2
A =
|aj | =
|t| |bj | = |t| B
X
X
X
2
C =
aj bj =
tbj bj =
t |bj | = tB
so
AB
2
|C|
2
2
= |t| BB = |t| B 2
2
2
= |tB| = |t| B 2
2
and AB = |C| . (-1pt by somehow assuming that t is real)
Note that by switching aj and bj and noting that (bj ) = 0 can be written
as (bj ) = t (aj ) with t = 0, (4) can be equivalently expressed “equality
holds if and only if one of (aj ) or (bj ) is a (complex) multiple of the other.”
Let’s prove this. (Not required, unless the result is somehow written in
this form.)
3
Related documents